LSAT and Law School Admissions Forum

Get expert LSAT preparation and law school admissions advice from PowerScore Test Preparation.

 emilysnoddon
  • Posts: 64
  • Joined: Apr 22, 2016
|
#26319
Is answer choice B saying that if there was practically no fishing before the ban then this could not possibly explain why there was a rebound in Quapaw Lake? I was confused about this question and chose answer choice D.
User avatar
 Jonathan Evans
PowerScore Staff
  • PowerScore Staff
  • Posts: 726
  • Joined: Jun 09, 2016
|
#26366
Emily, this is a great example of a causal flaw in a weaken question. In other words, the flaw or problem with the argument involves a purported connection between the occurrence of two events, that the occurrence of one somehow leads to the occurrence of the other.

In the conclusion of this argument, the author contends that "the ban on fishing is probably responsible for the rebound in the fish population at Quapaw Lake." Alarm bells should be ringing in your head.

Any time causal language (such as "responsible for") is present in an argument or an answer choice, you must ask yourself, "Do I really know that the ban caused the rebound? Could there be some other cause? Could it just be coincidence? Etc." Causal connections open up a big can of possible flaws.

Now focus on what you want the correct answer to do. In your prephrasing, you should note that you're looking for something that will tell you that the ban on fishing is NOT probably responsible for the rebound in the fish population at Quapaw lake. Perhaps there is another reason for the rebound. Perhaps the ban just had nothing to do with anything. Ideally, you want something that addresses a missing link between what happened at Quapaw lake before and after the ban. If you'll pardon the pun, the Highwater Lake analogy is mostly a red herring.

You need an answer choice that attacks the validity of the conclusion through the flaw, through the missing link, which in this case is causal.

(D) leaves far too many loose ends and just garbles the situation. It does not address the situation at Quapaw lake and doesn't address the causal connection.

The credited response (B) tells you something about the situation before and after the ban on fishing at Quapaw lake. If before the ban there was practically no fishing at Quapaw lake, what difference does it make to the fish population if fishing is banned? Not much at all, clearly. Ask yourself about the effect this answer choice has on the conclusion. Well, if there's not much fishing that ever happened and the population has gone down and now up, there must be something else going on here that's leading to the rise in the fish population! Do we know what it is? No! Do we care? No! All we know is that with this new information it is far less likely that the ban on fishing caused the increase in fish population post ban.

I hope this explanation helps clear up your question. If you need further assistance with this problem, please reply and let me know.
 LSAT2018
  • Posts: 242
  • Joined: Jan 10, 2018
|
#46771
For answer (D) Several other lakes in the Pawpaw mountains have recently had increases in their fish populations, I noticed that this is similar to common answers in other weaken/strengthen questions. How would we distinguish whether answers like these are irrelevant (because it isn't focusing on the subject in the stimulus) or weaken/strengthen answers (saying that a similar situation happened, which applies to the argument)?

Sometimes, I find myself eliminating them because they seem irrelevant, when they aren't and vice versa.
 Adam Tyson
PowerScore Staff
  • PowerScore Staff
  • Posts: 5153
  • Joined: Apr 14, 2011
|
#47133
You're right that we often see correct answers to causal questions that address parallel situations, LSAT_2018. When that happens, though, the answer still needs to provide us with enough information to classify it as one of our standard causal answers. If this answer were instead to say "Several other lakes in the Pawpaw mountains have recently had increases in their fish populations despite fishing still being allowed there," then perhaps we would classify that answer as one of "the effect is present where the cause is absent," and that might indeed weaken the argument. As it is, though, this answer only gives us half of that information - somewhere else that may be similar to the place we are looking at also had the same effect. Did it have the same cause, or did it not? Since we can't know, that is where this answer falls flat.

When reviewing answer choices to a weaken question for a causal argument, ask yourself "does this answer suggest an alternate cause? Does it show the cause is present when the effect is not, of the effect is present when the cause is not? Does it suggest that the causal relationship may be reversed? Does it undermine any data that the author relied on in arriving at their conclusion?"

Answer B is a little subtle. If you look at the cause not as being a fishing ban, exactly, but as "less fishing", then you can see that answer B shows a case of the cause being absent (if there wasn't much fishing to begin with, the ban wouldn't lead to much less fishing), but the effect is still present (fish numbers increasing). The ban probably isn't the cause, because it wouldn't have changed the situation much, if any. Something else has to be the cause of the rebound.

You could also look at answer B as a type of attack on the data, which is the least common of our 5 causal attacks. The author making this claim about the ban being the cause has to have assumed that there was a substantial enough amount of fishing going on before the ban to have impacted the fish population. If that assumption about the data turns out to be false, that weakens the causal claim that relied on that assumption.

Take another look at answer D, and look for more than just a similar situation. Look for whether that situation gives you one of the five things that weakens a causal argument. I think you'll find that it only gives you half the picture, and that will make it much easier to reject.
 coolnick
  • Posts: 1
  • Joined: Sep 01, 2018
|
#57045
I had a similar issue to the previous posters regarding B and D, and ended up choosing D.

I eliminated B because I read the answer as leaving open the possibility that there was indeed some fishing happening at Quapaw Lake. Even if there was very little fishing the moratorium meant there was no fishing, meaning that the fact remains that there was a change in the amount of fishing that was happening at Quapaw Lake due to the moratorium, and why wouldn't I assume that this small amount of change could still be the cause that led to the effect? If the answer choice stated that there was an "insignificant amount of fishing before the ban" then I think I would have jumped on that right away. It seems I, and maybe others, interpreted the phrase "practically no" incorrectly due to its common usage in day-to-day life ("I practically ate that whole pizza" - meaning I ate a whole lot of that pizza, but didn't actually eat ALL of it) whereas this answer seems to use it to say that the amount of fishing prior to the ban had no practical effect on the fish population (but doesn't this still require an extra assumption? Who am I to say whether the fishing had an effect or not? There was SOME fishing, now there's none).

D seemed much more clearly to weaken the argument by suggesting that there is some other factor causing an overall trend of increasing fish populations in the PawPaw mountain lakes. There would have to be a lot of assumptions to come up with a concrete reason that this weakens the moratorium argument, but should I need a concrete reason? Doesn't just the suggestion that this could just be some overall trend in the region's lakes i.e. another possible cause, be enough to put a check in the weaken column?

Taking a totally rigid definition of the wording "practically no" in answer choice B, I'd definitely agree that it weakened the argument to a much greater degree than answer choice D would have in my interpretation. It's amazing how we need to be able to compress this amount of thought and analysis into 70 seconds in order to answer these questions efficiently.

TL,DR: This test is nasty.
 Rachael Wilkenfeld
PowerScore Staff
  • PowerScore Staff
  • Posts: 1358
  • Joined: Dec 15, 2011
|
#57353
Hi Nick,

I agree that there's room for a small amount of fishing in "practically no" fishing. There could maybe be a person or two that pull a fish or two every month. However, the difference between "practically no" fishing and a complete moratorium on fishing wouldn't really address the recovery from a serious decline. We'd expect to see a fairly significant population increase to recover from a serious decline, one that wouldn't really be explained by the slight difference between practically no fishing and no fishing.

We just don't know enough for answer choice (D) to be correct. We have no idea if the fishing moratorium was region wide for example. Or if there was an environmental factor in common. Or if maybe someone is spiking the lakes with new fish. Answer choice (D) doesn't clearly link into the stimulus to weaken the explanation that it's the moratorium that increased the fish population because we don't know if that moratorium was present in the other lakes mentioned in answer choice (D).

Hope that helped!
Rachael
 ronibass
  • Posts: 15
  • Joined: Jun 18, 2019
|
#65908
I chose A. Is A wrong because it is talking about Highwater lake when the answer choice should be addressing something about Quapaw lake?
 Malila Robinson
PowerScore Staff
  • PowerScore Staff
  • Posts: 296
  • Joined: Feb 01, 2018
|
#65953
Hi ronibass,
Answer A is incorrect because it would require quite a bit more information to determine its relevance. For example it notes that Lake H is in an area highly susceptible to acid rain, but we don't know whether Lake Q is similarly situated. We would also need to know whether acid rain had anything to do with fish reproduction/re-population. Without knowing those things we can't be sure whether Answer A would weaken the argument.
Hope that helps!
-Malila
 Juanq42
  • Posts: 29
  • Joined: Jul 21, 2019
|
#67846
Hi,

I read through the forum posts but I remain unable to identify answer D as incorrect for October 2015 Digital LR1 9.

I understood the stimulus as followed-
Within the PawPaw mountain region, 2 lakes (Q and H) had a decline in their fish population. The causal relationships I identified are:
w/ Ban >>>> Q fish population increases
W/O Ban >>> H fish population decreases
Conclusion: Ban >>> Q fish population rebound


I also categorized answer choice D as "effect w/o cause." and based off previous questions, I recognized this answer choice as being correct.

I am trying to understand answer choice B by also considering a hypothetical timeline, but any other input would be appreciated:
The first day of 2000, when they started the ban at Lake Q, Q's fishing population began to increase. Lake H's fishing population received no ban, and their fishing population began to decrease.

10 years later, someone makes the claim that "the ban on fishing is probably responsible for the rebound in the fish population at Quapaw Lake"

Answer choice B would counter by stating-
In the last day of 1999, when there was no ban, there was no fishing.

How does this weaken the argument?
 James Finch
PowerScore Staff
  • PowerScore Staff
  • Posts: 943
  • Joined: Sep 06, 2017
|
#67867
Hi Juan,

You're correct in what the stimulus is saying, but you're making an assumption about (D) that we don't actually know: do those other lakes have bans or not? If yest, then this would be a strengthen answer, same cause leading to the same effect; if no, it's a weaken, as you'd have the effect with no cause. But we don't know, it gives us only the effect, so we can't use it to infer anything about the stimulus.

(B) is tricky, but does weaken the argument by attacking the assumption that fishing was causing the initial declines, making it so that banning it couldn't possibly have caused the rebound. Instead, there must be another cause, weakening the argument by implying the necessity of an alternate cause.

Hope this clears things up!

Get the most out of your LSAT Prep Plus subscription.

Analyze and track your performance with our Testing and Analytics Package.